6 votos

En la mecánica cuántica, ¿por qué $\langle T\rangle=\frac{\langle p^2 \rangle}{2m}$ en lugar de $\langle T\rangle=\frac{\langle p \rangle^2}{2m}$ ?

Soy un novato en la lectura de la mecánica cuántica desde "Inroduction to Quantum Meachanics" de Griffiths y en las primeras páginas del libro el autor define:

$$\langle x\rangle =\int_{-\infty}^{\infty} x|\Psi(x,t)|\,dx = \int_{-\infty}^{\infty} \Psi^* (x)\Psi \,dx,$$

$$\langle v\rangle = \frac{d}{dt}\left(\langle x\rangle\right)= -\frac{i\hbar}{m}\int_{-\infty}^{\infty} \Psi^*\frac{\partial\Psi}{\partial x} \,dx,$$

$$\langle p\rangle = m\langle v\rangle= -i\hbar\int_{-\infty}^{\infty} \Psi^*\frac{\partial\Psi}{\partial x} \,dx,$$

por lo que para mí el autor parece estar trabajando con las expectativas, lo que tiene mucho sentido para mí. Luego busqué en Google la expresión de la energía cinética y esperaba encontrar eso:

$$\langle T\rangle=\frac{\langle p \rangle^2}{2m},$$

pero en cambio, parece que

$$\langle T\rangle=\frac{\langle p^2 \rangle}{2m}.$$

¿Por qué? No entiendo qué pasó en el caso de la energía cinética. ¿Por qué el autor no trabaja ahora con el momento esperado en el caso de la energía cinética esperada? ¿Puede acaso mostrarme una derivación de $\langle T\rangle $ y lo que es más importante, la explicación de por qué se hace así ? En el libro, el autor dice que en general:

$$\langle Q(x, p)\rangle = \int \Psi^*Q(x, \frac{\hbar}{i}\frac{\partial}{\partial x})\Psi\,dx,$$

con el asesoramiento de que cada $p$ debe sustituirse por $\frac{\hbar}{i}\frac{\partial}{\partial x}$ al calcular la expectativa de interés. Sin embargo, el por qué de esto era un poco inexistente.

1 votos

La primera respuesta aquí puede ayudarte: physics.stackexchange.com/questions/424800/

0 votos

Mientras tanto, lo explicaré con la intuición más que con las matemáticas. El impulso tiene una dirección, la energía cinética no. Se puede tener un momento medio de $0$ pero una energía cinética media no nula. Realizar la media del cuadrado del momento soluciona esto.

0 votos

@MartinC. Gracias, de hecho comprobé esa respuesta antes pero no se me abría realmente por desgracia :/

8voto

Frank Waller Puntos 61

Si lo piensas, esto realmente no se reduce a la QM y sólo depende de cómo se tomen los promedios. La QM sólo entra en juego si realmente quieres calcular esos promedios dado el vector de estado del sistema.

Sabemos que $T=\frac{p^2}{2m}$ por lo que la media de esto es entonces $$\langle T\rangle=\left\langle\frac{p^2}{2m}\right\rangle=\frac{\langle p^2\rangle}{2m}$$

Ya que, en general, $\langle p^2\rangle\neq\langle p \rangle^2$ Aquí es donde terminamos.

Si se quiere encontrar este valor utilizando la base de la posición, entonces invocamos a QM: $$\langle T\rangle=-\frac{\hbar^2}{2m}\int\Psi^*\frac{\partial^2}{\partial x^2}\Psi\ dx$$

Esto se debe a que en la base de la posición, el $P^2$ operador es $-\hbar^2\frac{\partial^2}{\partial x^2}$ .

0 votos

Gracias, me ha ayudado un poco, pero sigo sin entenderlo al 100%. La razón por la que creo que me confunde, es porque en el caso de la velocidad, utilizamos la posición esperada. En el caso del momento, utilizamos la velocidad esperada. Pero en el caso de la energía cinética NO utilizamos el momento esperado. ¿Por casualidad conoces algún libro donde se explique esto en detalle? :)

0 votos

@jjepsuomi ¿Qué parte? ¿Cómo funcionan los promedios con funciones de otras variables, o cómo calculas esos promedios? Ten en cuenta que el núcleo de tu pregunta (por qué $\langle p^2\rangle$ en lugar de $\langle p \rangle^2$ ) no se trata de QM

2 votos

@jjepsuomi La energía cinética utiliza el valor de la expectativa del momento al cuadrado. La respuesta muestra explícitamente cómo funciona esto: $\langle T\rangle=\left\langle\frac{p^2}{2m}\right\rangle=\frac{\langle p^2\rangle}{2m}$ .

8voto

Oeufcoque Penteano Puntos 331

¿Por qué?

Para concretar, veamos un ejemplo específico para el que $\langle T \rangle \ne \frac{\langle P \rangle^2}{2m}$

Consideremos el caso de que tenemos una partícula con vector de estado (trabajando en 1D para simplificar)

$$|\psi\rangle = \frac{1}{\sqrt{2}}\left(|+p\rangle + |-p\rangle\right)$$

donde $p \ne 0$ y $P\,|\pm p\rangle = \pm p\,|\pm p\rangle$ (son eigenkets del operador de momento).

Evidentemente, el expectativa El valor del impulso es

$$\langle P\rangle = \langle\psi|P|\psi\rangle = \frac{1}{2}\left(+p -p\right) = 0$$

Esto se debe a que la medición del momento tiene la misma posibilidad de producir $+p$ y $-p$ .

Sin embargo, una medición de la energía cinética puede sólo rendimiento

$$T = \frac{(\pm p)^2}{2m} = \frac{p^2}{2m}$$

y así

$$\langle T \rangle = \frac{p^2}{2m} \ne \frac{\langle P \rangle^2}{2m} = 0$$

i-Ciencias.com

I-Ciencias es una comunidad de estudiantes y amantes de la ciencia en la que puedes resolver tus problemas y dudas.
Puedes consultar las preguntas de otros usuarios, hacer tus propias preguntas o resolver las de los demás.

Powered by:

X